Вы находитесь на странице: 1из 49

1.

As parent and the same time a teacher, which of the following will you do to show your cooperation to the
PTA project in your school to be financed with the proceeds of the sales of the school canteen where food
prices are little bit higher?
a. Bring food for you and your children, but always make it a point to buy in the school canteen.
b. buy all your food in the school canteen but request for a discount
c. bring food enough for you and your children but do not eat in the school canteen
d. buy all your food from the school canteen even if you cannot afford to do every day

2. According to the existentialist, every person in the same predicament and has the same possibilities.
What does this imply?
a. Every person must go to school.
b. Every person must go through the same from school.
c. Every person must earn a college degree.
d. Every person must be given access to education.

3. You are very much interested in a quality professional development program for teachers. What
characteristic should you look for?
a. prescribed by top educational leaders
b. dependent and availability of funds
c. required for the renewal of professional license
d. responsive to identified teachers’ need

4. The singing of the National anthem is an offshoot of philosophy of ____.


a. Nationalism c. naturalism
b. pragmatism d. Socialism

5. The environment in order to facilitate, learning must be interactive. Which of the following best typifies this
kind of environment?
a. the child goes out and discovers for himself some rock or fossil.
b. the child listens to a lecture on the fossils given by the teacher
c. the child summarizes the section on the fossils in his science textbook
d. The child copies a list of facts concerning fossils on the blackboard.

6. A teacher is said to be a “trustee of the cultural and educational heritage of the nation and is under
obligation to transmit to learners such heritage”. Which practice makes the teacher fullfill such obligation?
a. Use interactive teaching strategies.
b. Use the latest educational technology
c. Observe continuing professional education.
d. As a class, study the life of Filipino heroes.

7. For more efficient and effective management of schools as an agent of change, one proposal is for the
DepEd to cluster remote stand-alone schools under one lead school head. Which factor has the strongest
influence on this proposal?
a. Psychological c. Geographical
b. Historical d. Social8. What does the acronym EFA imply for school?
a. the acceptance of exclusive schools for boys and for girls
b. the stress on the superiority of formal education over that of alternative learning system
c. practice of inclusive education
d. the concentration on formal education system.
9. Which Republic Act provides government assistance to the students and teachers in private school?
a. RA 7784 c. RA 7836
b. RA 6728 d. RA 7722

10. the authoritarian setting in the Filipino home is reinforced by a classroom teacher who:
a. encourage pupils to ask questions
b. prescribes what pupils should do
c. is open to suggestions
d. ask open ended question

11. Who among believes that learning requires disciplined attention, regular homework and respect for
legitimate authority?
a. essentialist c. Progressivist
b. existentialist d. Recontructionist

12. The Constitutional provision on language has the following aims EXCEPT:
a. to make the regional dialect as auxiliary media of instructions in regional school
b. to maintain English as second language
c. to make Filipino the sole medium of instruction
d. to make Filipino the national language and medium of instruction and communication

13. The tendency to emphasize o much on school beautification to the detriment of pupils performance
illustrates the:
a. Filipino’s lack of seriousness
b. Filipino’ lack of reflection
c. Filipino’s sense of humor
d. Filipino’s love for “porma”

14. Which I Not a characteristics of democratic discipline?


a. child has opportunity to express his/her opinion
b. child given punishment is related to the misdeed
c. child understand the meaning of rules
d. child obeys blindly

15. Who among the following reasons stressed the processes of experience and problem solving?
a. Dewey c. Socrates
b. Aristotle d. Plato

16. The wide Acceptance of ‘bottom up” management style has influenced the schools to practice which
management practice?
a. Exclusion of politicians from the pool of guest speaker during graduation exercises
b. prescription of what ought to be done from the central Office
c. Involvement of students, parents, teachers, and community in the school planning
d. Allowing schools to do what they think is best.

17. Which characterizes the perfectionist type of students?


a. does not volunteer or initiate
b. Give up easily
c. Rarely complete tasks
d. often anxious fearful or frustrated about the quality of work
18. The failure of independent study with mot Filipino students may be attributed to students’
a. unpreparedness of schooling
b. ambivalence
c. high degree of independence
d. high degree of dependence on authority

19. Despite opposition from some school officials, DepEd has continuously enforced the “no collection of
fees” policy during enrolment period in public schools. Is this policy in accordance with EFA goals?
a. No, it violates the mandate of quality education
b. Yes, it somewhat eliminates gender disparities
c. Yes, it supports equitable access to basic education

20. Which of the following measures should a teacher do to a principal whom she would like to file a case of
sexual harassment w/o violating the relationship of the teacher to her superior?
a. present the case before competent authority and prepare to prove the charge
b. writ an anonymous letter to a higher school official to denounce the superior
c. call a parent-teacher meeting and denounced the superior
d. encourage the other teacher and students to hold a demonstration to oust the superior

21. To earn units for promotion, a teacher pays her fees but does not attend class at all. Does this constitute
professional growth?
a. Not immediately but yes after promotion.
b. It depends on the school she is enrolled in.
c. No, it simply earning MA units for promotion.
d. Yes, just enrolling in an MA program

22. Kounin claims that “with-it-ness” is one of the characteristics of an effective classroom manager. What is
one sign of with-it-ness?
a. Giving attention to students who are having difficulty with school work.
b. seeing only one portion of the class but intensively
c. knowing where instructional materials are kept
d. Aware of what’s happening I all parts of the classroom.

23. A student collapsed in her social studies class. It was found out that he did not eat her luch. What
principle is shown in the situation?
a. Psychological need c. Ecological need
b. Physiological need d. Safety need

24. Which techniques (s) enable (s) a teacher to identify and eventually assists students which interpersonal
difficulties?
a. Anecdotal record c. Cumulative record
b. Personal inventory d. Sociogram

25. Which is considered the “brain” of the computer?


a. CPU c. Video screen
b. Software d. keyboard

26. Zero standard deviation means that:


a. The student scores are the same
b. 50% of the scores obtained is zero
c. More than 50% of the score obtained zero
d. Less than 50% of the scores obtained zero

27. Which of the least authentic mode of assessment?


a. Paper-and-pencil test in vocabulary
b. Oral performance to assess students to socialize students spoken communication skills
c. Experiments in science to assess in the use of scientific methods
d. Artistic production for music or art subject

28. What must a teacher do to ensure orderly transitions between activities?


a. Allow time for the students to socialize in between activities
b. Have the materials ready at the start of the activity
c. Assign fewer exercises to fill the allotted time.
d. Wait the students who lag behind

29. When teacher tries to elicit clarification on a student response or solicits additional information, which of
these should he use?
a. Directing c. Structuring
b. probing d. Cross examining

30. A negative discrimination index means that:


a. More from the lower group answered the test items correctly
b. the items cloud not discriminate between the lower and upper group
c. more from the upper group answered the test item correctly
d. Less from the lower group got the test item correctly

31. ‘’When more senses are stimulated, teaching and learning become more effective.” What is an
application of this principle?
a. Appeal to students’ sense of imagination
b. use multisensory aids
c. make your students touch the instructional material
d. Use audio visual aids because the eyes and the ears are the most important senses in learning.

32. I combined several subject areas in order to focus on a single concept for interdisciplinary teaching.
What strategy did I use?
a. Reading-writing activity c. Lecture
b. Thematic instruction d. problem-centered learning

33. A teacher is a facilitator of learning and of the development of the youth. Which practice NOT keeping
with this role a facilitator?
a. Considers the multiple intelligence of learners
b. Humiliates misbehaving pupils
c. Dialogs with parents and with other member of the community
d. Keeps himself abreast with educational trends

34. Which one indicates a teacher’s genuine enthusiasm and pride in teaching?
a. Sticking to teaching for the moment that there are no better offers
b. Telling everyone that he went to teaching for there was no other choice them.
c. Engaging himself in continuing professional education
d. Belittling the re-numeration one gets from teaching35. Teacher A teaches English as a second Language.
She uses vocabulary cards, fill-in-the-blank sentences, dialogues, dictation and writing exercises in teaching
a lesson about grocery shopping. Based on this information, which of the following is a valid conclusion?
a. The teacher wants to make her teaching easier by having less talk.
b. The teacher is emphasizing reading and writing skills
c. The teacher is teaching in variety of ways because not all students learn in the same manner.
d. The teacher is applying Bloom’s hierarchy of cognitive learning

36. To teach the democratic process to the pupils, Biag Elementary School decided that the election of class
officers shall be patterned after local elections. There are qualifications set for candidates, limited period for
campaign and rule for posting campaign materials, etc. Which of the following did the school use?
a. Symposium c. Panel discussion
b. Simulation d. Debate

37. Which statement applies when scores distribution is negatively skewed?


a. The mode is lesser than the median c. The mode and median are equal
b. The median is higher than the mode d. The mean is lesser than the mode

38. In the Preamble of the Code of Ethics of Professional Teachers, which is not mentioned about teachers?
a. Duly Licensed professionals c. LET passer
b. Possess dignity and reputation d. With high moral values

39. What does a skewed score distribution mean?


a. The scores are concentrated more at one end or the other end
b. The mode, the mean and the median are equal
c. The mean and the median are equal
d. The scores are normally distributed

40. Teacher C, a reading teacher, advised her class to “read between the lines.” What does she want her
pupils to do?
a. Make an educated guess
b. Determine what is meant by what is stated
c. Apply the information need
d. Describe the characters in the story

41. On which constitutional provision is the full or partial integration of capable deaf and blind students in the
classroom based? The provision on
a. Providing citizenship and vocational training to adult citizen
b. Protecting and promoting the right of all citizen to quality education
c. Academic freedom
d. Creating Scholarship for poor and deserving students

42. Teaching the cognitive, psychomotor and affective domains is based on the concept that the learner is a:
a. Moral and feeling being c. Thinking, feeling and acting being
b. Material and an acting being d. Spiritual and material being

43. Which of the different types of the test covers and wide variety of objective?
a. True-false c. Matching type
b. Multiple choice d. Essay
44. Teacher wants to compare 2 concepts. With which technique can be accomplish this best?
a. K-W-L technique c. Spider web
b. Venn diagram d. Fishbone diagram

45. When necessary conditions are present, the use of inductive method is preferred because
a. There is greater active participation on the part of the pupils
b. It gives the teacher more time to rest
c. It needs only few instruction materials
d. Academic time is used wisely

46. Teacher B teaches his/her students that pleasure is NOT the highest good. What teacher teaches is
contrary to which philosophy?
a. Empiricism c. Hedonism
b. Realism d. Idealism

47. Which best indicates the effectiveness classroom activities?


a. The laughter and employment of students
b. The application of concept learned in daily life
c. The utilization of varied techniques and approaches
d. The variety instructional material used

48. Which is the most obvious and familiar way of reporting variability?
a. Range of scores c. Standard error of the mean
b. Standard deviation d. Distribution of raw scores49. If the teacher’s pattern in questioning consists of
calling on a student then asking the question
a. All students may be encouraged to participate
b. the student called to answer may be able to think well of his answer
c. The rest of the class may just dictate the answer
d. The rest of the class may not engage themselves in thinking of the answer50. Teacher L says: “ If it is
billiard that brings students out of the classroom, let us bring it into the classroom. Perhaps, I can use it to
teach Math.” To which philosophy does Teacher L adhere?
a. Reconstructionism c. Existentialism
b. Essentialism d. Progressivism

51. A child refuses to obey orders or displays negativism as a development trait. How may you best handle
him?
a. take every opportunity to praise him for every positive attitudes display
b. detain him after office hours for him to do what he has been ordered to do
c. insist on compliance to the same degree required of pupils
d. avoid giving him orders or if you do and he objects take the order back

52. Which term refers to the collection of student’s products and accomplishment for a period of evaluating
purposes?
a. Portfolio c. Anecdotal record
b. Observation report d. Diary

53. Which practice is an offshoot of B.F. Skinner’s theory of operant conditioning?


a. ensuring mastery of language c. use of scaffolding
b. use of programmed instruction d. considering multiple intelligence
54. In testing, which of the following is referred to as cultural bias?
a. Test items are more familiar in some culture
b. Some culture do better on tests than others
c. Test will show who is more cultured
d. Cultured people do better on tests

55. You have a pupil who is so talkative, naughty and aggressive that he is burden to the entire members of
the class. How would you remedy this problem?
a. Call the parent for a dialogue c. Reprimand him always
b. Report the case to the principal d. Talk to him seriously

56. Which of the following is the best situation wherein you can balance responsibility and accountability?
a. A teacher paid on an hour basis, takes her time with the subject matter till end of the period
b. A teacher paid on an hour basis, teaches as much as she could for duration of the period
c. A teacher paid on an hour basis, spends most of the time on the latest gossips in showbiz
d. A teacher paid on an hour basis, entertain her students w/ stories till the end of the period.

57. Section 5, article XIV, of the Constitution states that academic freedom shall be enjoyed in
a. Public assemblies’ c. All level of learning
b. State college and Universities d. All institutions of higher learning

58. Which of the following should a teacher do if she cannot pay the monthly instalment of an appliance she
got from a department store in their own?
a. Reject any notice of demand for payment to make the impression that she will be receive
b. Move to another neighbourhood to escape payment
c. Inform the manager of the store personally and make a satisfactory arrangement of payment on or before
the due date of payment
d. Offer to return the used appliance to the store on the condition that she will be refunded on the monthly
instalment she paid.

59. Freud expounded that there is a period when young boys experience rivalry with their father for their
mother’s affection. This is
a. Oedipus Complex c. Achilles syndrome
b. Electra complex d. Cassandra syndrome

60. Education is a lifelong process. This simply means that education-


a. May take place formally or informally to enable the individual to grow
b. May take place anywhere and anytime the individual so desire
c. Is a Continuous process of experiencing and recognizing experience
d. Take place in the school where the individual is expose, self-contained experiences.

61. The tendency to imitate elders is very strong in the early childhood stage. Teacher should therefore be
very good-
a. Counselors c. Role model
b. Disciplinarians d. Facilitator of Learning62. Teacher Z always checks on entry knowledge and skills before
she proceeds to her new lesson. On which principle is Teacher Z practice grounded?
a. Effective teaching proceeds from the concrete to abstract
b. learning increase when the learning is relevant
c. Attention is essential for learning
d. New learning builds on previous learning

63. Which of the following is the best time for a teacher to set up routine activities that will contribute to
effective classroom management?
a. As soon as the student have established
b. Daily as the start of the session
c. During his homeroom day
d. On the very first day of school

64. Teacher R wants to develop his students’ creativity. Which type of questions will be MOST appropriate?
a. Synthesis questions c. “What if….” questions
b. Fact questions d. Analysis question

65. You want your student to answer the questions at the end of the reading lesson: ‘’ What did I learn
today?”, ‘What still puzzles me?” What did I enjoy, hate, accomplish in class today?” , “How did I learn from
the lesson?’’ , Which of the following are you asking them to do?
a. Work on an assignment c. Work on a drill
b. make entry journal d. Apply what they learn

66. If the student is encourage to develop himself to the fullest and must satisfy his hierarchy of needs, the
highest need to satisfy according to Maslow is ___________.
a. psychological need c. belongingness
b. self-actualization d. safety need

67. In Social Studies class, Teacher I presents a morally ambiguous situation and asks students what they
would do. On whose theory is Teacher I’s technique based?
a. Bandura c. Kohlberg
b. Freud d. Erickson

68. Based on Freud’s theory, which operate/s when a student strikes a classmate at the height of anger?
a. Ego c. Id and Ego
b. Id d. Erickson

69. The Computer r for score in Math and Science is 0.92. What does this mean?
a. Math score is positively related to Science score
b. The higher the Math score, the lower the Science score
c. math score is not in any way related to Science score
d. Science score is slightly related to Math score

70. Which type of the tests is most appropriate if Teacher Y wants to measure students’ ability to organize
thoughts and ideas?
a. Short answer c. Alternative response
b. Essay d. Multiple Choice

71. If Teacher wants to measure her students’ ability to discriminate, which of these is an appropriate type of
the test item as implied by the direction?
a. “Outline the Chapter on The Cell.”
b. “Summarize the lesson yesterday.”
c. “Group the following items according to shape”
d. “State a set of principles that can explain”

72. A test item has a difficulty index of 0.89 and a discrimination index of 0.44. What should the teacher do?
a. Reject the item c. Make it a bonus item
b. Retain the item d. Make it a bonus item and reject it.

73. What can be said of Arielle who obtained a score of 75 out of 100 items in a Grammar objective test?
a. She performed better than 25% of her classmate
b. She answered 25 items in the test correctly.
c. Her rating is 75.
d. She answered 75% of the test items correctly.

74. The criterion of success in Teacher D’s objective is that “the pupils must be able to spell 90% of the
words correctly.” Ana and 24 others in the class spelled only 40 out of 50 words correctly while the rest
scored 45 and above. This means that Teacher D ___________.
a. attained her lesson objectives
b. did not attained her lesson objective because of the pupils lack of attention
c. failed to attain her lesson objective as far as the 25 pupils are concerned
d. attained her lesson objective because of her effective spelling drill

75. For her discussion of the topic from various perspectives, it is BEST to hold __________.
a. brainstorming c. debate
b. Symposium d. panel discussion

76. Availment of the Philippine Education Placement Test for adult and out-of-school youth is in the support
of the government’s educational program towards _______________.
a. Relevance c. Equitable access
b. Quality d. Quality and Relevance

77. Which MDG goal is related to the state’s goal for equality education?
a. MDG 1 c. MDG 3
b. MDG 2 d. MDG 4

78. Which illustrates vicarious punishment?


a. We feel so bad to the classmate who is punished for being tardy so we convince him go to school on time.
b. Out of comparison, we volunteer to get punished in the place of friend
c. We charge to experience our being punished
d. See someone who get punished for habitual tardiness. In effect, we are fees likely to be tardy

79. Which objective in the effective domain is the lowest level?


a. To accumulate examples of authencity
b. To support view against abortion
c. To respond positively to a comment
d. To formulate criteria for honestly

80. In instructional planning, it is necessary that the parts of the plan from the first to the last have:
a. Symmetry c. Conciseness
b. Coherence d. Clarity
81. The military training requirements among students in the secondary and tertiary levels can be traced as
a strong influence of the
a. Spartans c. Chinese
b. Romans d. Athenians

82. To promote effective practice, which guideline should you bear in mind? Practice should be _________.
a. Difficult for students to learn a lesson
b. Arrange to allow students to receive feedback
c. Done in an evaluative atmosphere
d. Take place over a long period of time

83. By what name is direct instruction or Socratic Method also known?


a. questioning method c. indirect method
b. Morrison method d. mastery learning

84. Which does NOT belong to the group of alternative learning systems?
a. Multi-age grouping c. non-grade grouping
b. Multi-grade grouping d. graded education

85. In the social-constructivist perspective, the child is given a task but he/she cannot accomplish it alone
without the assistance of adults or peers around him/her.
a. ZPD c. Social learning theory
b. the Socratic method d. peer tutoring

86. Which term applies to the search for related literature by computer access of data bases on disc kept in
libraries?
a. On-line research c. Compact disc computer research
b. Manual research d. computer research

87. The whole child concept of education is contrary to the:


a. Academic essentials approach to education
b. Progressivist approach to education
c. Focus of humanistic approach to education
d. The naturalistic approach to education

88. Here is a lesson objective in literature: “Using the six description of the elements of a good short story
identify in writing the six elements in the short story by O. Henry with complete accuracy.” Applying Robert
Mager’s principle on writing performance objectives, which is the criterion measure?
a. “ the six element in the short story by O. Henry with complete accuracy
b. “identify in writing”
c. “ with complete accuracy”
d. “ using the six description of the element for a short story”

89. With Bloom’s cognitive taxonomy in mind, which objective is in the highest level?
a. To rate a project along relevance, originally and craftsmanship
b. to state the assumption that underlies the given statements
c. To state generalization from the data given
d. To write a paragraph that observe s unity from the data given
90. Which refer to a single word or phrases that tell the computer to do something with a program or file?
a. Computer program c. Computer language
b. Password d. Command

91. What is the mean of the score distribution: 4, 5,6,7,8,9,10?


a. 7.5 c. 6
b. 8.5 d. 7

92. Which is the teaching approach for a kindergarten makes real world experiences of the child the focal
point of educational stimulation?
a. Situation approach c. Electric approach
b. Traditional approach d. Montessori approach

93. Which is a characteristic of an effective classroom management?


a. Coercive c. Imposed
b. Reactive d. Preventive

94. Which can run counter to the encouragement you to give your students to ask question?
a. Knitted eyebrow when a question is raised
b. eye to eye contact
c. radiant face
d. an encouraging hand gesture

95. Which one can help students develop the habit of critical thinking?
a. Blind obedience of authority
b. Asking convergent questions
c. Asking low level questions
d. A willingness to suspend judgement until sufficient evidence is presented

96. In instructional planning, which among the three: unit, plan, course plan, lesson plan is most specific?
a. Course c. Resource
b. Unit d. Lesson

97. Teacher Mar wants to develop in her pupil’s comprehension skills. What order of skills will she develop?
I. Literal comprehension III. Critical Evaluation
II. Interpretation IV. Integration
a. II-III-IV-I c. I-II-III-IV
b. III-IV-I-II d. IV-III-II-I

98. If the student thinks about hi/her thinking, he is involved in the process called:
a. Higher-order thinking c. Creative thinking
b. Critical thinking d. Metacognition

99. The use of drills in the classroom is rooted on Thorndike’s law of:
a. Readiness c. Exercise
b. Effect d. Belongingness

100. The following are some drill techniques, EXCEPT


a. Challenging students to be above the level of the class
b. Asking pupils to repeat answers
c. Giving short quiz and having students grade papers
d. Assigning exercises from a workbook101. The first American teachers in the Philippines were
a. Soldiers c. Missionaries
b. Graduate of the normal school d. Elementary graduates

102. Which questioning technique promotes more classroom interaction?


a. Focusing in convergent question
b. Calling on the student before asking question
c. focusing on divergent question
d. Asking rhetorical question

103. Teacher V wants to check prior knowledge of his pupils about water pollution. She writes the main topic
water pollution in the center of the chalkboard and encircles it. Then, she asks the pupils to provide
information that can be clustered around the main topic. Which technique did the teacher employ?
a. Vocabulary building c. Semantic Mapping
b. Demonstration d. Deductive teaching

104. Who were the Thomasites?


a. The soldiers who doubted the success of the republic educational system to be set in the Philippines
b. The first American teacher-recruits to help establish the public educational system in the Philippines
c. The first religious group who came to the Philippines on board the US transports Thomas
d. The devotee to St. Thomas Aquinas who came to evangelize Filipinos

105. To build a sense of pride among Filipino youth, which should be done?
a. Re-study our History and stress on our achievements as a people
b. Re-study our history from the perspective of our colonizers.
c. Replace the study of folklores and myths with technical subjects
d. Set the study of local history

106. “Specialization is knowing more and more about less and less. Then it is better to be generalist”, claims
Teacher F. On which philosophy does Teacher F learn?
a. Essentialism c. Perenialism
b. Progressivism d. Existentialism

107. Teacher Lei gives his students opportunity to be creative because of his conviction that much learning
results from the need to express creativity. On which theory of teacher Lei conviction anchored?
_______________ theory
a. Behaviorist c. Cognitive
b. Associationist d. Humanist

108. Which teaching activity is founded on Bandura’s social learning theory?


a. Questioning c. Modeling
b. Inductive reasoning d. Interactive reasoning109. I like to develop the synthesizing skills of my students.
Which one should I do?
a. Ask my student to formulate a generalization from the data shown in the graphs
b. Direct my students to point out w/c part of the graph are right and w/c part is wrong
c. Ask my students to answer the questions beginning with “What if….”
d. Tell my students to state data presented in the graph.

110. To reach out to clientele who cannot be in the classroom for one reason or another, which of the
following was established?
a. Informal c. Pre-school education
b. Special education (SPED) system d. Alternative learning delivery

111. Nicolle enjoys games like scrabble, anagrams, and password. Which type of intelligence is strong in
Nicolle?
a. Interpersonal intelligence
b. Logical and mathematical intelligence
c. Linguistic intelligence
d. Spatial intelligence

112. Teacher Cora observes cleanliness and order inter classroom to create a conductive atmosphere for
learning. On which theory is her practice based?
a. Psychoanalysis c. Behaviorism
b. Gestalt psychology d. Humanistic psychology

113. Which learning principle is the essence of Gardner’s theory of multiple intelligence?
a. Almost all learners are linguistically intelligent
b. Intelligence is not measured in one form
c. Learners have different IQ level
d. Learners have static IQ

114. Based on Edgar Dale’s Cone of Experience, which activity is farthest from the real thing?
a. Watch a demo c. Listen to a lecture
b. View image d. Attend exhibit

115. Which is a proactive management practice?


a. Tell them that you enforce the rules on everyone, no exception
b. Set and clarity your rules and expectations on Day 1
c. Punish the misbehaving pupils in the presence of their classmate
d. Stress on penalty for every violation

116. For integrative purposes, which method/approach should be used?


a. Metacognitive approach c. Thematic approach
b. Cooperative approach d. Constructive approach

117. Which quotes goes with a proactive approach to discipline?


a. “An ounce of prevention is better than a pound of cure.”
b. “Do not make a mountain of a mole.”
c. “Walk your talk”
d. “ Do not smile until Christmas”

118. What does the principle of individual differences require teacher to do?
a. Give less attention to gifted learners.
b. Provide for a variety of learning activities.
c. Treat all learners alike while teaching.
d. Prepare modules for slow learners in class.

119. Teachers are encouraged to make use of authentic assessment. Which goes with authentic
assessment?
a. De-contextualized drills
b. Unrealistic performances
c. Answering multiple choices test items
d. Real word application of lesson learned

120. I’d like to test whether a student knows what a particular word means. Which should I ask the students
to do?
a. Give the word a tune then sing it
b. Define the word.
c. spell the word and identify its part of speech
d. Give the etymology of the word.

121. With SMART lesson objectives in the synthesis level in mind, which one does NOT belong to the
group?
a. Formulate c. Build
b. Organize d. Dissect

122. Who asserted that children must be given the opportunity to explore and work on different materials so
that they will develop the sense of initiative instead of guilt?
a. Kohlberg c. Maslow
b. Erickson d. Gardner

123. Teacher Zen uses direct instruction strategy. Which will she first do?
a. Presenting and structuring c. Independence practice
b. Guided student practice d. Reviewing the previews day’s work

124. The free public elementary and secondary education in the country is in the line with the government
effort to address educational problems of _____________.
a. Productivity c. Access and equality
b. Relevance and Quality d. Effectiveness and efficiency`

125. Here row score in the quiz 97 95 85 83, 77, 75, 50 10, 5 ,2, 1, to get a picture of the group’s
performance, which measure of central tendency is MOST reliable/
A. Median
B. Mean
C. Mode
D. non.it is best to look at the individual scores

126. Which statement about standard deviation is CORRECT?


a. The lower the standard deviation the more spread the scores are.
b. The higher the standard deviation the more spread the scores are.
c. The higher the standard deviation the less spread the scores are
d. It is a measure of central tendency.

127. which is the teachers’ professionalization act?


a. R A 7836
b. R A 4670
c. R A 7722
d. R A 9293
128. The state shall protect and promote the right of all citizens to quality education at all level. ’’Which
government program is in support of this?
a. Exclusion of children with special needs from the formal system
b. Free elementary and secondary education
c. Deregulated tuition fee hike
d. Re-introduction of the NEAT and NSAT

129. The K-12 curriculum is the otherwise called as


a. Basic Education Curriculum
b. Revitalized Basic Education Curriculum
c. Enhanced Basic Education Curriculum
d. Extended Basic Education Curriculum

130. The Filipino learners envisioned by the Department of Education in the light of K-12 curriculum is
a. Technologically literate and holistically developed Filipinos
b. Functionally literate and holistically developed Filipinos
c. Scientifically advanced and values oriented Filipinos
d. Nationally oriented and internationally competitive Filipinos

131. Principle tells her teachers that training in the humanities is most important To which education
philosophy does he adhere?
a. Existentialism
b. progressivism
c. Essentialism
d. Perennialism

132. Tony exhibits fear response to freely roaming dogs but does not show fear wen a dog is on a leash or
confined in a cage. Which conditioning process is illustrated?
a. Generalization c. Discrimination
b. Acquisition d. Extinction

133. Behaviour followed by pleasant consequences will be strengthened and will be more likely to occur in
the future. Behaviour followed by unpleasant Consequences will be weakened and will be less likely to be
repeated in the future. Which one is explained?
a. Freud’s psychoanalytic theory
b. Thorndike’s law of effect
c. B.F. Skinner’s operant conditioning theory
d. Bandura’s social learning theory

134. Theft of school equipment like T.V., computer, etc. by people in the community itself is becoming a
common phenomenon. Which does this incident signified?
a. Prevalence of poverty in community
b. Inability of school to hire security guards
c. Deprivation of Filipino schools
d. Community’s lack of sense of co-ownership

135. The main purpose of compulsory study of Constitution is to ____________.


a. Develop students into responsibilities, thinking citizen.
b. Acquaint students with the historical development of the Phil. Constitution.
c. Make constitutional experts of the students
d. Prepare students for law-making

136. With which goals of educational institution as provided for by the constitution is the development of
work skills aligned?
a. To develop moral character c. To teach the duties of citizenship
b. To develop vocational efficiency d. To inculcate love of country

137. Parents admit that SMS has an effect on _______________.


a. Communicating through guessing c. Shortening messages effectively
b. Ability to guess words correctly d. Ability to spell words correctly

138. In computer based instruction, which tool can help you revise your short stories essays and other
written work?
a. Word processing c. Database
b. Spread sheet d. Desktop publishing

139. Which is closest to the real human digestive system for study in the classroom?
a. Model of the human digestive system.
b. Drawing of the human digestive system on the board.
c. The human digestive system projective on an OHP
d. Drawing of the human digestive system on the page of the textbook.

140. Here is a question: “Is the paragraph a good one? Evaluate.” If broken down to simplify, which is the
BEST simplification?
a. Is the paragraph a good one? Why or why not?
b. Why is the paragraph a good one? Prove.
c. If you ask to evaluate something, what do you do? Evaluate the paragraph.
d. What qualities of a good paragraph? Does the paragraph have these qualities?

141. I want my student to have mastery learning of a basic topic. Which can help?
a. Problem Solving c. Lecture Method
b. Drill d. Socratic Method

142. What is the mastery level of a school in a 100-item test with a mean of 55?
a. 42% c. 45%
b. 50% d. 55%

143. Who stressed the idea that students cannot learn if their basic needs are not first met?
a. Thorndike c. Wertheirmer
b. Maslow d. Bandura

144. A person who had painful experiences at the dentist’s office may become fearful at the mere sight of
the dentist’s office building. Which theory can explain this?
a. Attribution theory c. Social learning
b. Classical conditioning d. Operant conditioning

145. I want the fast learners in my class to do self-directed learning. What strategy will I use?
a. Collaborative learning c. Meta-cognitive
b. Social dialogue d. Problem based learning
146. For which lesson objective will you use the direct instruction method?
a. Appreciate Milton’s Paradise Lost
b. Use a microscope properly
c. Distinguish war from aggression
d. become aware of the pollutants in the environment

147. To encourage introspection, which teaching method is MOST appropriate?


a. Cognitive c. Process
b. Reflective d. Cooperative

148. With indirect instruction in mind, which does NOT belong to the group?
a. Experimental method c. Lecture-recitation method
b. Inductive method d. Discovery method

149. A master teacher, the resource speaker in an in-service training presented the situated learning theory
and encouraged her colleagues to apply the same in-class. Which did she NOT encourage her colleagues to
do?
a. Apprenticeship c. Learning as it normally occurs
b. Decontextualized teaching d. Authentic problem solving

150. Teachers are encouraged to make use of authentic assessment. Which goes with the authentic
assessment?
a. De-contextualized drills
b. Unrealistic performances
c. Answering multiple choice test question
d. Real word application of lesson learned

151. Teacher Jay discovered that her pupils are weak in comprehension. To further determine in which
particular skill(s) her pupils are weak; which test should Teacher Jay give?
a. Standardized Test c. Placement Test
b. Aptitude Test d. Diagnostic Test

152. Which will be the most authentic assessment tool for an instructional objective on working with and
relating with people?
a. Writing articles on working relating to people
b. Organizing a community project
c. Home visitation
d. Conducting mock election

153. Teacher Lorelle does norm-referenced interpretation of score. Which of the following does she do?
a. She uses a specified content as frame reference.
b. She describes the class performance against a set level of mastery.
c. She compares individual student’s scores with other student’s scores.
d. She describes what should be every individual student’s performance.

154. You practice inclusive education. Which of these apply to you?


I. You accept every student as full and valued member of the class and school community.
II. Your special attention is on learners with specific learning or social needs
III. You address the needs of the class as a whole within the context of the learners with specific learning or
social needs.
a. II only c. I only
b. I and II d. I and III

155. What principle is violated by overusing the chalkboards, as though it is the only education technology
available?
a. Isolated use c. Variety
b. Flexibility d. Uniformity

156. Which statement applies CORRECTLY to Edgar Dale’s “Cone of Experience”?


a. The father you are form the base, the more direct the learning the experience becomes.
b. The father you are form the bottom, the more direct the learning experience becomes
c. The closer you are to the base, the more indirect the learning experience become.
d. The closer you are to the base; the more direct learning experience becomes.

157. I want to teach concepts, patterns and abstractions. Which method will be MOST appropriate?
a. Discovery c. Direct instruction
b. Indirect instruction d. Problem solving

158. Teacher Ann, an experienced teacher, does daily review of past lessons in order to ____.
a. Introduce a new lesson
b. Reflect on how presented the previous lesson
c. Provide his pupils with a sense of continuity
d. Determine who among his pupils are studying

159. To nurture students’ creatively, which activity should be a AVOID?


a. Ask “what if…” questions
b. Ask divergent thinking questions
c. Emphasize the need to give right answer
d. Be open to “out-of-this-world” ideas

160. After reading and essay, Teacher B wants to help sharpen her student’ ability to interpret. Which of
these activities will be most appropriate?
a. Drawing conclusion c. Getting the main idea
b. Making interference d. Listing facts separately from opinion

161. Under no circumstances shall a teacher be prejudiced nor discriminatory against any learner, says the
Code of Ethics. When is a teacher prejudice against any learner?
a. When he makes a near sighted pupils set at the front
b. When he considers multiple intelligences in the choice of teaching strategies
c. When he makes a farsighted pupil sit at the back
d. When he refuses a pupil with slight physical disability in class

162. Which learning activity is most appropriate if teacher’s focus I attitudinal change?
a. Role play c. Exhibit
b. Fieldtrip d. Game

163. The mode of a score distribution is 25. This means that


a. There is no score of 25.
b. Twenty five (25) is the score that occurs most
c. Twenty five is the average of the score distribution
d. Twenty five is the core that occurs least

164. The following characterize a child-centered kindergarten EXCEPT ______.


a. Focus on the education of the whole child
b. Importance of play in development
c. Extreme orientation an academic
d. Emphasis on individual uniqueness

165. The last year of pre-service education is student teaching. Student teaching is classified as
____________.
a. Role playing c. Field trip
b. Simulation d. Demonstration

166. Which term refers helping a colleague grow professionally?


a. Technology transfer c. Facilitating
b. Peer monitoring d. Independent study

167. What primary criterion should guide a teacher in the choice of instructional devices?
a. Novelty c. Appropriateness
b. Cost d. Attractiveness

168. Which of the following is considered peripheral device?


a. Printer c. CPU
b. Keyboard d. Monitor

169. Which questioning practice will promote more class interaction?


a. Asking rhetorical question
b. Rejecting wrong question
c. Focusing on convergent question
d. Asking the question before calling on a student to answer

170. Research on Piagetian tasks indicates that thinking becomes more logical and abstract as children
reach the formal operations stage. What is an educational implication of this finding?
a. Expect hypothetical reasoning for learners between 12 to 15 years of age.
b. Learners who are not capable of logical reasoning from ages 8 to 11 behind in their cognitive
development.
c. Engage children in analogical reasoning as early as preschool to train them for higher order thinking skills
(HOTS)
d. Let children be children

171. A mathematics test was given to all Grade V pupils to determine the contestants for the Math Quiz Bee.
Which statistical measure should be used to identify the top 15?
a. Percentage score c. Quartile Score
b. Mean percentage Score d. Percentile Score

172. How can you exhibit expert power on the first day of school?
a. By making them feel you know what you are talking about
b. By telling them the importance of good grades.
c. By reminding your students your authority over them again and again
d. By giving your students a sense of belonging and acceptance

173. The following are sound specific purposes of questioning EXCEPT ___________.
a. To stimulate learners to ask questions
b. To call attention of an attentive student
c. To arouse interest and curiosity
d. To elicit answers from students

174. Which is NOT a sound purpose for asking questions?


a. To remind students of a procedure
b. To probe deeper after an answer is given
c. To encourage self-reflection
d. To discipline a bully in class

175. A student passes a book report poorly written but ornately presented in a folder to make up for the poor
quality of the report of the book report content. Which Filipino trait does this practice prove?
a. Art over science c. Substance over “porma”
b. Art over academic d. “Porma” ove substance

176. Which material consists of instructional units that cater to varying mental levels pupils?
a. Plantilla c. Multi-grade materials
b. Multi-level materials d. Minimum learning competencies

177. In K-W-L technique K stands for what the pupils already knows, W for what he wants to know and L for
what he
a. Learned c. Failed to learn
b. he likes to learn d. Needs to learn

178. Self-rating and follow-up conference with teacher benefit both teacher and student. The following are
for the students EXCEPT
a. heighten student’s defensiveness
b. helps the student develop increased skill in self-assessment
c. helps the students recognize his progress toward the instructional objectives
d. helps the student diagnose his particular strengths and weaknesses

179. Which one should you do if a parent of one of your failing pupils asks you to tutor her daughter in
consideration of a certain amount of money which badly need?
a. Accept the offer but do tutoring outside office hour
b. direct the parent to another tutor and make internal arrangement with the tutor for commission
c. accept the offer with discount
d. advise the parent to look for another tutor

180. Teacher C does not personally agree with one school policy. What is the professional thing for him to
do?
a. lead a campaign against the abolition of that school policy
b. make honest effort to understand, support, and carry out the school policy even if he does not personally
agree
c. be indifferent about it sa she exerts effort to understand
d. defy the policy because in conscience he cannot agree
181. A teacher should not be a slave of his lesson plan. This means that
a. A lesson plan must be followed by a teacher no matter
b. A teacher must be ready to depart from her lesson plan if she remembers something more interesting that
what she earlier planned
c. A teacher must be willing to depart from her lesson plan if students are interested in something other than
her interested lesson.
d. Teacher is the best lesson plan designer

182. To provide for individual differences how is curriculum designed?


a. Minimum learning competencies are included
b. Realistic and meaningful experiences are provided
c. Some degree of flexibility is provided
d. Social skills are emphasized

183. Which is the TRUE foundation of the social order?


a. Strong, political leadership c. Equitable distribution of wealth
b. The reciprocation of rights and duties d. Obedient citizenry

184. A test consists of a graph showing the relationship between age and population. Following it is a series
of true-false items based on the graph. Which type of test does this illustrate?
a. Laboratory exercise c. Performance
b. Problem solving d. Interpretive

185. Which curricular move served to strengthen spiritual and ethical values?
a. Integration of creative thinking in all subject
b. Reducing the number of subject areas into the skills subject
c. Introduction of Values Education as separate subject area
d. Re-introducing Science as all subject in Grade 1

186. A teacher notices glaring wrong pronunciation of vowel sounds among her students necessitating more
practice. Which of the following activities would be of help?
a. Dictionary c. Assignment
b. Review d. Drill

187. Teacher A is observed to be a bit aloof from the children of the Aetas. She justifies her action by saying
“We are about human; we cannot like every pupil.” Is she acting ethically?
a. Yes, because Teacher A is not yet engaged in a destructive behaviour.
b. Yes, persons have their own peculiarities and we expect that we cannot like everybody in the same way
that not everybody can like us.
c. No, under no circumstances shall a teacher be prejudices against any learner
d. No, unless she has extraordinary reason for being aloof to the Aetas.

188. Which of the following teaching practices should be AVOIDED?


a. Using “put down” strategy c. Asking more divergent questions
b. Using multiple response strategy d. Asking more evaluative questions

189. Which of the following embodies the operation “return to the basics”?
a. National Secondary Achievement Test
b. New Secondary Curriculum
c. New Elementary School Curriculum
d. National Elementary Achievement Test

190. Which of the principle of development is manifested in the saying “as a tree is bent, so shall it grow that
of, “a person’s action reflect the training he received as a child?”
a. The early formative years are more crucial than later development
b. Development is determined by the environment or how one is nurtured
c. Development is determined by heredity or one’s inherent nature
d. It is never too late to teach a child new ways
1. Which Republic Act provides government assistance to students and teachers in private education?
a. RA 7836 c. RA 6728
b. RA 7784 d. RA 6675
2. In the preamble of the CODE of ethics for Professional Teachers, which of the following teacher
descriptions is included in the Code of Ethics for Professional Teachers?
a. A. With satisfactory teaching performance
b. Passed the licensure exams for teachers
c. Duly licensed professional
d. Persons of dignity and reputation
3. Which stage according to Erickson is the stage of trust and mistrust?
a. Young adulthood c. Adolescence
b. Early childhood d. Infancy
4. In the history of education system, including that of the Philippines, which system was first and last
remained a partner of other systems of education?
a. Formal b. Non-formal c. Informal d. Pre-school
5. Who among the following subscribe to the theory that the good, the true and the beautiful are universally
valid in all places at all times?
a. Realists and idealists
b. Realist and existentialists
c. Pragmatists and idealists
d. Pragmatists and realists
6. The 1987 Constitution mandates the state to encourage among others self-learning, independent and out-
of-school study programs particularly those that respond to community needs. Which is an off shoot of this
mandate?
a. Institutionalization of early childhood care and development
b. Principal empowerment program
c. Multi-grade teaching
d. Practices of community-based learning
7. Authors claim that education is given greater in the 1987 Philippine Constitution than in the past
Constitution. Which article provision on education is found only in the 1987 Constitution?
a. Support to teacher education
b. Provision on assigning the highest budget priority to education
c. The establishment of the integrated system of education
d. The creation of CHED
8. According to Hurlock, studies of children’s play revealed that toy play reaches its peak during the
a. Middle childhood years
b. Early childhood years
c. Babyhood
d. Late childhood
9. In a DECS memorandum issued last 1998, the new name for multi-grade class is:
a. A.Friendly school c. Paaralan ng bahay
b. School-of-the-future d. Escuela nueva
10. The student-centered curriculum belongs to which group of educators?
a. Reconstructionist-Essentialist
b. Progressivist, Perennialist
c. Perennialist, Essentialist
d. Progressivist, Reconstructionist
11. Daniel Goleman talks about emotional intelligence. Which of the following characterize a student with a
high degree of emotional intelligence?
a. Sensitive to points of view and feeling of others
b. Prodded by others
c. Obsessed with achieving at any cost
d. Unable to delay gratification
12. The history of curriculum development includes the hidden curriculum approach. Which is the content
emphasis of the hidden curriculum?
a. Students experiences and activities
b. Implicit processes and social norms
c. Student needs and interest
d. Introspection and choice
13. Teacher T is concerned with conceptual matters since reality is mental teacher T’s thinking is quite-
a. Realistic c. Existential
b. Pragmatic d. Idealistic
14. The authoritarian setting in the Filipino home is reinforced by a classroom teacher who-
a. Asks open –ended questions
b. Encourages pupils to ask questions
c. Prescribes what pupils should do
d. Is open to suggestions
15. A number of researchers found that the effects of maternal employment on children’s achievement are-
a. Hardly established
b. Positive and negative
c. Fully established
d. Negative
16. Which one correctly describes the phenomenon of latchkey children?
a. They turn to empty homes after school either to sit by the television or to roam the streets
b. They are truant school children
c. They are engaged in child labor
d. They are engaged in child labor
17. Teacher Q regards the student as a spiritual entity and as part of the larger spiritual universe. To what
philosophy does teacher Q subscribe?
a. Idealism c. Realism
b. Existentialism d. Pragmatism
18. The 1987 Constitution mandates the state to assign the highest budgetary priority to-
a. Health c. Social work
b. Education d. Defense
19. Moral development in early childhood is characterized by-
a. Questioning rules
b. Willful disobedience of rules
c. Acting out of conviction
d. Acting without knowing why they do so
20. According to Piaget, by adolescence boys and girls must have reached what stage?
a. Concrete operations stage
b. Sensorimotor stage
c. Formal operations stage
d. Pre-operational stage
21. Ruth dances well. She can figure out how something works or how to fix something that is broken,
without asking for help. Based on Garden’s theory of multiple intelligence under what intelligence is she
strong?
a. Musical
b. Logical-mathematical
c. Spatial intelligence
d. Kinesthetic intelligence
22. Who among the following believes that learning requires disciplined attention regular homework, and
respect for legitimate authority?
a. Reconstructionist c. Essetialist
b. Perennialist d. Progressivist
23. Which developmental task is expected of the adolescent according to havughurst?
a. Achieving new and more mature relations with age-mates of both sexes
b. Skilled games
c. Getting started in an occupation
d. Learning to get along with age-mates
24. Mr. Roy wants his pupils to be creative. Which of the following will he REFRAIN from doing?
a. Heighten the student’s sense of unusual
b. Develops students’ ability to recognize and analyze problems and relationships
c. Encourage sustained attention in an area of interest
d. Develop solely skill mastery
25. The creation of Student’s Loan Fund to give equal opportunity to all persons who desire to pursue higher
education is the essence of:
a. RA 6014 c. RA 6655
b. RA 7836 d. RA 7784
26. In his/her teaching, Teacher D moves from particular instances to tentative generalizations that are
Subjected to further verification . Teacher D engages himself/herself in:
a. Intuition
b. Inductive logic
c. Deductive logic
d. Philosophical analysis
27. Which of the correct statement of emotional intelligence is based on Daniel Goleman’s theory?
a. Emotional intelligence is feeling approximately and effectively
b. Emotional intelligence id being nice to people
c. Emotional intelligence is giving free rein to feeling
d. Emotional intelligence changes less considerably than IQ through life.
28. Which of the following is not a subject-centered curriculum?
a. Perennialist c. Values-centered
b. Back-to-basics d. Subject area
29. Which program is assisted financially by the World Bank and OECF and is meant for the improvement of
elementary education in the SRA provinces?
a. Program for Decentralized Education
b. First Elementary Education Project
c. Kinesthetic intelligence
d. Second Elementary Education Project
30. Mr. Z, A fifty-five year old American citizen, even though very educationally qualified to be president an
educational institution in third country cannot be appointed as President because: He_____
a. Does not belong to a religious group
b. Is nearing retirement
c. Is not Filipino Citizen
d. Is not an official of a corporation
31. Which of the following is not a hazard to the mastery of developmental tasks?
a. Unfavorable social judgment
b. Bypassing of stage of development as a result of failure to master the tasks for that stage of development
c. Crisis when individuals pass from on stage to another
d. Inappropriate or impossible expectations
32. Teacher U emphasizes to her students the importance of deep personal reflection on one’s
commitments and choices. Teacher U subscribes to which Philosophy?
a. Realism c. Existentialism
b. Idealism d. Pragmatism
33. The constitutional provision on language has the following aims, Except:
a. To make Filipino the national language and medium of instruction and official communication
b. To make the regional dialect as auxiliary media of instructions in regional schools
c. To make Filipino the sole medium of instruction
d. To maintain English as a second language
34. Which periods in the life span of an individual are characterized by growth spurt?
a. Early childhood and adolescence
b. Prenatal period and early childhood
c. Early childhood
d. Pre-natal and puberty
35. The introduction of non-formal education in line with the Constitutional Provision on:
a. Protection of teachers
b. Promoting the rights of all citizens to make quality education accessible to all
c. Optional religious instructions
d. Promoting the rights of all citizens to quality education at all levels
36. Teacher X is often times frustrated. The students in her class hardly volunteer to recite and to do other
learning-related task. This is a proof of the Filipino’s:
a. Sense of humor
b. Lack of resourcefulness
c. Lack of reflection
d. Passivity and lack of initiative
37. The tendency to emphasize so much on school beautification to the detriment of pupil’s performance
illustrated the:
a. Filipino’s sense of humor
b. Filipino’s lack of reflection
c. Filipino’s love for “ Porma” at the expense of substance
d. Filipino’s lack of seriousness
38. A president body to study Philippines education created by virtue of Executive Order No. 46 during the
incumbency of DECS secretary Andrew Gonzales with an aim to study Philippine Education is the:
a. Presidential Commission on Educational Reform
b. Philippine Commission to Survey Philippine Education
c. Survey to outcomes of elementary Education
d. Commission on Philippine Education
39. Which is a danger signal of adolescent mal-adjustment?
a. Inappropriate body-build
b. Irresponsibility as shown in neglect of studies in favor of having a good time
c. Attraction to the opposite sex
d. Prolonged treatment as children
40. To democratize access to secondary education, public secondary education was made free. In whose
presidency was this implemented?
a. Ferdinand Marcos c. Corazon Aquino
b. Joseph Estrada d. Fidel Ramos
41. Which is not a characteristic of democratic discipline?
a. Child obeys blindly
b. Child understands the meaning of rules
c. Child is given punishment us related to the misdeed
d. Child has opportunity to express his/her opinion
42. The three A’s of happiness according to Hurlock are:
a. Adjustments, affection, altruism
b. Attitude, ability, adjustment
c. Acceptance, affection, and achievement
d. Affection, ability, attitude
43. Babyhood is often referred to as a “ Critical period” in the development of personality because:
a. At this time the foundations are laid upon which the adult personality structure will be built
b. Changes in the personality pattern take place
c. The brain grows and develops at such an accelerated rate during babyhood
d. At this time the baby is exposed to many hazards both physical and psychological
44. Who among the following stressed the processes of experiences and problem solving?
a. Plato c. Dewey
b. Hegel d. Aristotle
45. Teacher L believes that creation of knowledge in by way of the learners’ interaction with their
environment. Teacher L is more of:
a. An idealist c. An existentialis
b. A pragmatist d. A realist
46. To reach out clientele who cannot be in the classroom for one reason or another, which of the following
was established
a. Non-formal education
b. Informal education
c. Pre-school education
d. formal education
47. The encouragement of self-learning, independent, and out-of-school study programs as stated in the
1987 constitution has given rise to:
a. The implementation of open universities and distance learning programs
b. Institutionalization of early childhood care and development
c. The conduct of NEAT and NSAT
d. The clamor for a Grade VII
48. Which of the following statement on developmental tasks is wrong?
a. Failure to master developmental tasks at a certain developmental stage has far reaching consequences in
a person’s development
b. The mastery of development tasks is a result of physical maturation, societal pressure and individual’s
aspiration
c. There are essential skills expected to be acquired and mastered in each developmental stage
d. Retirees are not expected to work on mastering certain develop mental tasks
49. The 1987 Philippines Constitution states the following, Except:
a. The official languages of the Philippines are Filipino and English
b. The national language of the Philippines is Filipino
c. Filipino is the Tagalog of the Tagalog –speaking provinces
d. The government shall initiate and sustain the use of Filipino as a medium of official communication and as
a language of instruction in the education system
50. Which among the following has been said to play an irreplaceable role in the education of the young?
a. Community c. Church
b. School d. Home
51. For the drill method to be effective there should be:
a. Much practice on a few skills
b. Little practices on few skills
c. Much practice on many skills
d. Little practice on many skills
52. Which of the following classroom management practices runs counter to the reality therapy approach of
William Glasser?
a. The teacher asks a misbehaving student what he or she is doing
b. Teacher requires the students to prepare a plan in writing and sign it as means of increasing personal
motivation to maintain and fulfill the plan
c. The teacher evaluates pupil’s behavior as good or bad
d. Teacher assists the students in making realistic plans to change behavior
53. Which of the following does not apply to mastery learning?
a. It makes use of varied instructional time for different groups of students
b. It makes use of norm-reference tests
c. It is effective in teaching basic skills
d. It requires the use of carefully crafted instructional objective
54. The teacher’s role in problem solving methods is:
a. To test the conclusions
b. To set up the problem
c. To propose ways of obtaining the needed data
d. To help the learners what is it that is being solved
55. To nurture creativity in student, teacher should:
a. Vary the length and difficulty of question
b. Allow for one-minute wait-time
c. Ask convergent questions
d. Emphasize the necessity of giving right answers
56. The content of a good lesson plan is self-sufficient. This means that:
a. The content should match with student’s aptitude
b. The content should help students learn how to learn
c. The content should be broad and treated thoroughly
d. The content should be verifiable
57. Which one can NOT help individuals reduce their own fears about change?
a. Reflect on the extent of change in one’s life so far and how it has coped with
b. Identify shortfalls in skills or knowledge and take action to remedy them
c. Identify skills which will be useful in a new situation
d. Keep feeling of anxiety to themselves
58. The following are generating thinking skills, Except:
a. Connecting new ideas c. Classifying
b. Predicting d. Inferring
59. Mrs. Valdez wants to generates as many ideas as she can as the class is about to embark on a
community outreach program. Which of the following will she employ?
a. role playing c. brainwashing
b. brainstorming d. simulation
60. Which teaching practice will most likely decrease learner’s attention?
a. Questioning
b. Assessing learning as an integral part of instruction
c. Assessing learning as an integral part of instruction
d. Teaching by telling
61. For efficiency, if in the process of teaching a teacher realizes that too little has been planned, which of
the following may she NOT do?
a. Discuss possible problems of the new assignment.
b. Pose additional questions to explore various facets of the content
c. Drill the students on the major points of the lesson.
d. Give a quiz
62. Correct practice makes perfect. This maxim is based on Thorndike’s law of exercise and the finding that
reinforcement of a response increases the likelihood its occurrence. Whose research finding is the
underlined statement?
a. Skinner c. Palo
b. Bruner d. Lewin
63. Here is a lesson objective: Given a microscope and a slide, the students must be able to focus the
prepared slide. Applying Robert F. Mager’s principle, which does this lesson objective lack?
a. Criterion measure c. Performance
b. Condition d. time element
64. Which is a focusing thinking skill?
a. Identifying key concepts
b. Summarizing
c. Obtaining information
d. Clarifying through inquiry
65. In the use of television in the classroom, which of the following should be avoided?
a. The TV program lasts the whole class period.
b. There is a pre-viewing orientation
c. Select programs that match the learners’ level of interest and maturity
d. Life should be left on if students are to take notes
66. Based on Victor Lowenfield’s classification, which sequence in the developmental stages of children in
art is followed?
a. Scribbling stage, pre-schematic stage, schematic stage, dawning realism stage, pseudo-realistic stage
b. Scribbling stage, pre-schematic stage, schematic stage, pseudorealistic stage, dawning realism stage
c. Pre-schematic stage, schematic stage, scribbling stage, pseudorealistic stage, dawning realism stage
d. Pre-schematic, schematic stage, scribbling dawning realism stage, pseudo-realistic stage
67. In relation to teacher’s ratings research suggests the following, Except one:
a. Teachers favor self-evaluation over all other forms of evaluation
b. Peer and supervisory evaluation are not reliable
c. Student raters of teachers are more reliable and valid than other raters
d. Student work and test outcomes are supplementary sources for evaluating teachers
68. Suppose defined 3 levels of computer-assisted instruction: dialogue, tutoring, practice-drill. Which ones
are two higher levels in the correct order?
a. Tutoring and dialogue
b. Practices-drill and tutoring
c. Dialogue and tutoring
d. Practice-drill and dialogue
69. In what condition is the use of the lecture method appropriate?
a. Higher cognitive learning is sought
b. The information is not available
c. The subject matter is quite easy
d. Long term learning is desired
70. Which refers to the time when students learn at a maximum level?
a. Wait time c. Allocated time
b. Engaged time d. Academic learning time
71. What is a sign of the underachiever in the classroom?
a. Resist authority and carry on a power struggle with a teacher.
b. Holds back from class participant unless sure of self
c. Frustrated about quality of work
d. Minimum work output.
72. If after calling on a number of students, a teacher is unable to obtain the desired response, what should
teacher do
a. Ask leading questions
b. Re-teach parts of the lesson that need re-teaching
c. Probe student’s answers.
d. Rephrase the questions
73. For global competitiveness, a school must embark on a proactive change. Which one is a characteristic
of proactive change?
a. radical and inventive c. late in the game
b. imitative of others d. problem-driven
74. Which guideline on the use of the chalkboard should a teacher AVOID?
a. Establish routine uses for the chalkboard
b. While writing, proceed from right to left
c. Don’t talk to the chalkboard while writing on it
d. Limit your board writing to major ideas
75. To teach the democratic process to the pupils, Mabuhay Elementary School decided that the election of
class and school officers shall be patterned after local election of the class and school officers shall be
patterned after local election process. There are qualifications set for candidates, limited period for
campaign and rules for postingcampaign materials, etc. Which of the following did the school use?
a. Simulation c. Role playing
b. Philips 66 d. Symposium
76. Which is NOT a characteristic of preventive discipline.
a. Proactive c. Inventive
b. Anticipatory d. Reactive.
77. One criterion that has been proposed in the selection and organization of content in the development
section of a lesson plan is balance. When does lesson content possess balance.
a. The content should have practical application for the learners
b. The content is not cluttered by masses of more trivial content
c. The content should enable the learners to experience the broad sweep of content and give them the
opportunity to go deeper
d. The content should help student learns how to learn
78. The burnout malady gets worse if a teacher doesn’t intervene to change whatever areas he or she can
control. Which one can renew a teachers enthusiasm.
a. Engage in self-pity
b. Stick to the job
c. Judge someone else as wrong
d. Initiate changes in job.
79. Although learning can take place anywhere and anytime, the more systematic the teacher, the greater
the probability for success. This points out the need for teachers to:
a. Specify their objectives
b. Plan their lessons
c. Set their teaching priorities
d. Select their instructional materials
80. Below are teachers effective classroom management practices, Except One.
a. Teachers make sure students understand and follow rules and procedures
b. Disruptive behavior is handled every weekend
c. Teachers clearly establish consequences for not following rules
d. Teachers spend more time in the beginning of the year explaining and reminding students of rules.
81. A teacher’s summary of a lesson serves the following functions except;
a. It links the parts of the less
b. It makes provisions for full participation of the students
c. It clinches the basic ideas or concepts of the lesson
d. It brings together the information that has been discussed
82. Which ones are projected visuals?
a. Graphs on a book c. Realias
b. Models d. Slides
83. The lesson is on the pros and cons of capital punishment. Mr. Milan wants to do high level thinking and
to develop a view of capital punishment from a different perspective. Which technique will be most
appropriate?
a. Lecture c. Role playing
b. Simulation d. Panel discussion
84. Which is used to emphasize individualized instruction?
a. Tutorial group c. Inquiry group
b. Philips 66 d. Task group
85. Here a question: FROM THE DATA PRESENTED FORM GENERALIZATIONS THAT ARE
SUPPORTED BY THE DATA. Under what type of question does this test item fall?
a. Evaluate c. Divergent
b. Convergent d. Application
86. Ms. Estira cannot bring her pupils to the sea for a lesson on marine community. Which of the following
will be closest to an actual experience of marine community?
a. Motion picture on marine community
b. Description of marine community
c. Pictures of marine community
d. Taped lesson of marine community
87. Which can NoT help organizations reduces fears about change?
a. By training people in new skills
b. By keeping people informed about plans
c. By cladestine moves
d. By consultation
88. After a lesson in air pollution, teacher C gives each pupil mimeographed sheets, which contain
statements to which each is expected to react. Examples of these statements are. Do you in any way
contribute to air pollution? What solutions do you propose to minimize, if not eliminate, air pollution. Then the
students are grouped for sharing.
In this case what does the teacher make use of
a. Value sheet c. Rank ordering
b. Conflict story d. Contrived incident
89. Which orders consist of the goal-oriented instructional model?
a. Pre-assessment, specification of objectives, pre-assessment, instruction, evaluation
b. Specification of objectives, pre-assessment, instruction evaluation
c. Pre-assessment, specification of objectives, instruction, evaluation
d. Specification of objectives, evaluation, pre-assessment, instruction
90. Teacher B’s lesson was on abortion. She wanted to get her student’s maximum participation and
reaction. She showed slides on abortion accompanied by an audiotape that presented the reasons for and
against abortion. Teacher took a stand in favor of abortion and came up with all her justifications. At the end,
however, she made clear her stand on abortion. Which did the teacher use?
a. She made use of value clarifying discussion
b. She made use of contrived incident
c. She made use of simulation
d. She played the role of the devil’s advocate
91. In on of the pages of her reference material, Ms. Estrada finds the life cycle of a frog. Assuming that the
following are available to her, the quickest way to effectively present the life cycle of a frog to the class is by
way of
a. An overhead projector
b. An opaque projector
c. Describing the life cycle
d. A drawing on the chalkboard
92. Below are questions that must be considered in developing appropriate learning activities/ experience,
Except one
a. Do the experiences encourage pupils to inquire further?
b. Do the experiences save the pupils from learning difficulties?
c. Can be experiences profit the pupils?
d. Are the experiences in accordance with an increasing amount of learning?
93. Which order of the senses goes with an increasing amount of learning?
a. Sight, hearing, touch, taste and smell
b. Taste and smell, sight , touch, hearing
c. Taste and smell, touch, sight, hearing
d. Taste and smell, touch, hearing, sight
94. Which of the following statements on peer tutoring is true?
a. Teacher has time with more severe learning problems
b. The atmosphere is threatening
c. The social skills of pear tutors are not enhanced
d. the peer tutor’s achievement goes down
95. A lesson in Philippines history was presented by Mrs. Palaroan by making her class view a videotape on
EDSA revolution. The videotape which Mrs. Palaroan used in class points to what property of instructional
media?
a. Manipulative c. Fixative
b. Multi-sensory d. Distributive
96. Read the following teacher-student- situation. Teacher: Why is the process called photosynthesis?
Student: I don’t know. Which questioning technique should the teacher use?
a. Concept c. Clarification
b. Prompting d. Multiple response
97. Miss Lee’s objective is to focus student’s attention on and quickly create interest in a problem or
concept. She makes use of:
a. Task group c. Tutorial group
b. Panel discussion d. Philips 66
98. The content of the lesson must be feasible. This means that:
a. The lesson should be interesting to the student
b. The lesson must be within the capacity of the students
c. The teacher must considered the time needed and resources available
d. The lesson must be useful
99. Which questioning behavior is appropriate:
a. Not allowing a student to complete a long response
b. Repeating all student’s response
c. Allowing choral responses
d. Asking varied questions
100. Research on teacher effective practices has shown the following except:
a. Planning has title impact on student learning
b. Questioning strategies are ineffective monitoring technique
c. Teaching procedures on classroom routines early in the school year are essential
d. Directives should be few and best delivered in a casual manner
101. Teacher Y wants to measure student’s ability to organize thoughts and ideas. Which type of test is
most appropriate?
a. Objectives test
b. Limited response essay
c. Extended response essay
d. Short answer type of test
102. Which of the following refers to organized services specially designed to improve the adjustment of
students?
a. Group growth sessions c. Cumulative records
b. Test profiles d. Guidance program
103. Which tests survey existing problems of academic progress?
a. Achievement tests c. Problem checklist
b. Interest inventories d. Aptitude tests
104. Which of the following statements about guidance is NOT true?
a. Guidance is a continuous process
b. Guidance is concerned with “ Whole” student, not only with his or her intellectual and academic aspect
c. Guidance is telling a student what is best for him, and advising him about what he should do
d. Guidance is primary concerned with prevention rather than cure
105. Which of the following is NOT a guidance service?
a. Psychological testing
b. Counseling
c. Observation
d. Education placement
106. Which of the following tests is given at the end of instruction?
a. Summative c. formative test
b. Placement test d. diagnostic test
107. Special or exceptional students are such in certain aspects, Except for one:
a. Sensory abilities
b. Gender orientation
c. Mental characteristics
d. Communication abilities
108. Information from achievement tests can be used in the following, Except in:
a. Grading a student
b. Labeling pupils
c. Establishing proficiency
d. Giving feedback regarding effectiveness of learning
109. Which of the following occurs when the teachers have a general tendency to rate all individuals’
performance at
approximately the same position on the rating scale?
a. Personal bias error c. Logical error
b. Halo effect d. Severity error
110. The environment must be interactive to facilitate learning, which of the following situations is an
example of this?
a. The class copies a list of facts concerning the habitat of insects
b. The teacher lectures on the habitat of insects
c. The class goes out and discovers the habitat of insects
d. The teacher shows posters of the habitat of insects
111. When teachers, administrators and counselors come together to synthesize or coordinate in
interpreting data about a student for the purpose of a more intensive study, this is called
a. Longitudinal study c. Case conference
b. Case study d. Case analysis
112. Which type of test can help teachers predict the probability of success of students in certain areas or
endeavors?
a. Achievement c. Aptitude
b. Intelligence d. Personality
113. Which of the following correctly describes dyslexia?
a. A. It is a developmental expressive writing disorder
b. It is a developmental arithmetic disorder
c. It is a developmental reading disorder
d. It is a developmental articulation disorder
114. Free and compulsory education as mandated in the 1987 constitution holds true for_____________
a. Elementary and secondary education
b. Education in the primary grades
c. All children of school age
d. Grade VI pupils
115. Readiness pretests is give at the beginning of an instructional unit is a:
a. Summative tests c. Placement test
b. Formative test d. Diagnostic test
116. The distribution of the scores is negatively skewed. This means that:
a. Most of the score are low
b. Most of the scores are high
c. The score are concentrated on the left of the distribution curve and most of the score are low
d. The scores are concentrated on the left of the distribution curve
117. Aside from professional competence, what other important qualification must a teacher have for
guidance work?
a. Similar cultural background with the students
b. Personal qualities, such as warmth, open-mindedness, interest in people
c. Affiliation with a professional group
d. Several year of experience as teacher
118. Which of the following is meant to determine student performance at the beginning of instruction?
a. Diagnostic assessment
b. Placement assessment
c. Summative assessment
d. Summative assessment
119. Mean is to central tendency as standard deviation is to:
a. Discrimination c. Correlation
b. Level of difficulty d. Variability
120. The following characterize school guidance programs, Except one:
a. A part of every school activity
b. Separate from the general life of the school
c. A function sheared by all
d. Located in every part of the school
121. Which factor is most likely to make a school’s guidance program succeed?
a. Administrative support
b. The availability of the specialist
c. The favorable attitude of teachers toward guidance
d. Located in every part of the school
122. The results if this type of test serve as basis for remedial instruction. What is the type of test referred
to?
a. Prognotic test c. Diagnostic test
b. Speed test d. Achievement test
123. The difficulty index of a test item is 1. This implies that:
a. The test item must be moderate in difficulty because 50% got the item correctly and 50% got it wrongly
b. The test item must be very difficult because nobody got the item correctly
c. The test item must be neither difficult because nor easy because this depends on the ability of the
students
d. The test item must be very easy because everybody got the item correctly
124. In his conduct of item analysis Mr. Milanes discovered that a significant greater number from the lower
group of the class that from the upper group got item number 10 correctly. This implies that:
a. The test item is reliable
b. The test item has a negative discriminating power
c. The item has a positive discrimination power
d. The tests item is valid
125. Guidance is concerned with
a. All students, even if some seem not to have problems
b. Students who manifests undesirable behavior only
c. Underachieving students only
d. Students with emotional problems only
126. Which statement explains the primary focus of the non-directive approach in counseling?
a. It focuses in the person’s responsibility and capacity to discover more appropriate behavior
b. It places confidence in a person’s ability to deal with his or her needs through a realistic process
c. it helps confidence to develop a more rational to deal with his or her needs through a realistic process
d. it modifies behavior by providing appropriate learning conditions and experiences
127. An algebra test was designed to measure what the students learned at the end of the course. It was
also to predict success in future mathematics subjects. This algebra test functioned as-
a. Diagnostic and placement test
b. An aptitude test
c. An achievement test
d. An aptitude and an achievement test
128. Giftedness is a form of exceptionality. Students who fall under this category demonstrate high
performance in special areas, such as those below-
a. Athletic prowess c. General intellectual ability
b. Creative thinking d. Visual and performing arts
129. Which of the following is the least stable measure of central tendency?
a. Median and mode c. Median
b. Mode d. Mean
130. Which assessment is concerned with identifying learning difficulties during instruction
a. Summative assessment
b. Placement assessment
c. Formative assessment
d. Visual and performing arts
131. These are significant information about a student, gathered through the use of various techniques,
assembled, summarized and organized in such a way that they may be used effectively:
a. Cumulative records
b. Test profile
c. Personal inventories
d. Case studies
132. Research findings show that autism is
a. Either more prevalent among girls or among boys depending on their nationality
b. Equally prevalent among boys and girls
c. More prevalent among boys than among girls
d. More prevalent among girls than among boys
133. In interpreting test results which statements are true?
I. A raw score itself is meaningless but becomes meaningful once it is interpreted
II. An analysis of test results is useless without interpretation
III. Test result interpretation is possible without analysis
IV. The use of statistical techniques gives meaning to student’s scores
a. I, II, IV c. I, III, IV
b. I, II, III d. II, III, IV
134. This guidance service to help students carry out their plans and act on their choices so that they
become adjusted to their chosen field or career is called:
a. Counseling c. placement
b. information d. follow-up
135. The tendency for the rather to use only the lower end of rating scale in rating performance is referred to
as
a. Personal bias error c. Generosity error
b. Severity error d. logical error
136. What must be employed to continuously strengthen a school guidance program
a. Assignment of responsibilities
b. research
c. Evaluation
d. Program Planning
137. Which tests are given before instruction?
a. Placement c. Summative
b. Formative d. Achievement
138. The operation “Return to the basics” saw its embodiment in the _________
a. National Elementary Achievement Test
b. New Secondary Education Curriculum
c. National Secondary Achievement Test
d. New Elementary School Curriculum
139. The following are computed difficulty indices: 1, . 80, .50, .30 which one indicates the most difficulty
item?
a. 30 b. 50 c. 1 d. 80
140. Which of the following explains why the plateau phenomenon of ten found in human being does NOT
exist in animal learning experiments?
a. Animals are unable to profit from errors
b. The incentive in animal learning is constant
c. Animals have weak incentive in learning
d. Animals are unable to spurt in learning
141. All are components of remedial guidance in the classroom, Except:
a. Dealing with discipline problems
b. Dealing with poor study habits and skills
c. Dealing with underachievement
d. Dealing with career choices or vocational plans
142. Which method provides a more or less objective assessment of different aspects of an individual?
a. Home visits
b. Counseling interview
c. Observations
d. Standardized testing
143. Which of the following behavior indicates that a child has developed conventional maturity? The
behavior is based on _________
a. The desire to avoid severe physical punishment by a superior authority
b. Personal decisions based on his satisfaction
c. The expectations of the group or society in general to gain approval
d. Internalized ideals to avoid self-condemnation rather than social censure
144. A school guidance program should arise mainly out of:
a. The curriculum and other programs of the school
b. The school administrator’s directives from his/her perceptions
c. The students’ needs and the problems
d. The availability of guidance resources in the schools
145. A test in which the options are dependent upon a foundation of some sort such as graphical
representation, paragraph, and pictures is:
a. Contained-options test
b. Setting-and-options test
c. Structured-response
d. Stem-and-options test
146. In the scoring of essay tests, experts advise teachers to score all answer to an easy question before
going to the next question for which reason?
a. It reduce the chances that ratings will be influenced by a halo effect
b. It is more convenient
c. It is easier to apply the criteria more uniformly when considering only a single task at a time when going
from task to task for each student
d. Checking is faster this way
147. Whose performance does stanine of 5 represent?
a. Performance of the upper 40%
b. Performance of the upper 20%
c. Performance of the lower 20%
d. Performance of the middle 20%
148. In making and reporting observations of students’ behavior, which ones should be avoided?
a. Descriptions of student’s gestures and expressions
b. The names of other students with whom the one being observed interact
c. Opinions and judgment about students
d. Direct quotations of student’s statements
149. This is the stage when the learner become confused and starts to experience identity crisis. This is the
____________
a. Late childhood stage
b. Early childhood stage
c. Early adulthood stage
d. Adolescent stage
150. All are basic functions of psychological tests EXCEPT ONE.
a. For selection or admission
b. Identification of students who need special attention
c. For promotion or acceleration
d. Integration of life experiences and future directions of students
151. Upon what shall a teacher base the evaluation of the learner’s scholarship
a. Merit, learner’s scholarship
b. Merit, quality of performance
c. Merit, attendance
d. Quality of performance, learner’s scholarship
152. The following are some of what a professional teachers may NOT do EXCEPT:
a. Accept remuneration from tutorials other than what is authorized for such service
b. Make deductions from student’s scholastic ratings as a punishment
c. Base the evaluation of the learner’s work on merit and quality of academic performance
d. Inflict corporal punishment an offending learners
153. The value of education, deference to authority, and filial piety in the Philippines in an influenced of
a. Taoist tradition c. Buddhist tradition
b. Confucianist tradition d. Hindu tradition
154. Which statement regarding teacher’s ethical behavior is wrong?
a. A teacher may not fall in love with his/her students
b. Teachers shall support one another at all times when the best interest of the learners, the school, or the
profession at stake
c. A teachers may submit to the proper authorities any justifiable criticisms against an associate
d. It is every teacher’s responsibility to seek correctives for any unprofessional and unethical conduct of any
associate
155. Which is the first and foremost concern of a teacher? The interest and welfare of:
a. Learners
b. Colleagues
c. Parents
d. The teaching profession
156. The ideal society in the context of Plato is one where:
a. There is hope c. Justice reigns
b. Love prevails d. Faith matters
157. The survivors of a shipwreck are now packed in a lifeboat which may capsize unless partly deloaded.
Which of the following acts will be morally justifiable?
a. Pushing the young and old ones and leaving them behind to drown
b. Trying to save everyone as much as possible
c. Shooting some passengers and leaving them behind
d. Doing nothing and waiting for the boat to capsize
158. For the advocates of value clarification, values are
a. Universal c. Objective
b. Personal d. Unchanging
159. With regard to business, which does the Code of Ethics NOT say about teachers?
a. A teacher shall maintain a good reputation with respect to debts loans and other financial matters
b. No teachers shall be financially interested in any commercial venture involving textbooks and other school
commodities where he/she can exercise official influence
c. no teacher shall act as agent of textbooks and other school commodities where he/she can exercise
official influence
d. a teacher may not engage in any kind of business
160. The more consumeristic a person is, the more he/she gets attached to this material world, the farther
he is from the universal self, the more miserable he/she becomes. This is a teaching
a. Confucianism c. Hinduism
b. Taoism d. Buddhism
161. Who believed that there should be less reliance on the school, and the world of work?
a. Paolo Freire c. Paul Goodman
b. Ivan Illich d. Jean Paul Sartre
162. Human dignity is inherent in every person. This means that
a. The inner worth of a person depends on what he has
b. The inner worth of a person depends on how he/she looks
c. The inner worth of a person depends on what he can do
d. The inner worth of a person is in him/her she looks
163. Which of the following runs counter to the development of honesty and accountability?
a. Being vigilant about weights and measures used in the markets
b. Equal payment of government employee’s productivity pay
c. Payment of just wages to workers and employees
d. Transparent in operations of the barangay and
other organizations through public reporting
164. Which are said to be our wellspring of excellence?
a. Third largest English-speaking nation, tiger economy, inventor of people’s power revolution
b. Tiger economy, superior human resources
c. Tiger largest English-speaking nation, superior human resources, inventor of people’s power revolution
d. Tiger economy, the inventor of people’s power revolution, superior, human resources
165. Which among these goals for change was proposed by the moral Recovery Program?
a. A sense of justice and outrage over its violation, a sense of national pride, a sense of seriousness
b. A sense of national guide, a sense of seriousness, sense of common good
c. A sense of national guide, a sense of seriousness, sense of national justice and outrage over its violation
d. A sense of national guide, a sense of the common good, a sense of justice and outrage over its violation
166. The code of ethics for teachers states that school officials shall encourage and attend to the
professional growth of all teachers. According to the Code school officials can do this by:
a. Giving them due recognition for meritorious performance by allowing them to participate in conferences
and training programs recommending teachers for promotion
b. Recommending teachers for promotion, organizing teachers in a professional organization
c. Allowing them to participate in conferences and training programs
d. Organizing teachers into a professional organization, giving them due recognition for meritorious
performance and recommending teachers for promotion
167. Which thrust on value formation is intended to help the students identify and be aware of their values?
a. Analysis c. Value inculcation
b. Value clarification d. Moral development
168. The idea that there should be less reliance on the school and greater use of education potential of the
community and the world of work is the essence of:
a. Community-based education
b. Formal education
c. Multi-grade teaching
d. Mobile teaching
169. It is said that the big drama development is played out in small scenes. Which one does not serve as a
further explanation of this statement?
a. Ordinary things done by ordinary people are the solid building blocks on which a nation develops
b. Details can make a significant difference
c. Much of our personal development and progress of the nation depends on how extraordinary well we do
the ordinary things
d. Let us give attention to major things because they are the ones that matter
170. Which of the following characteristics does not apply to Plato’s just society?
a. The artists are creative
b. The leaders are wise
c. The soldiers are courageous
d. The working class are temperant
171. Martin Buber’s “ dialogic principle” requires as restructuring of society which can be achieved by:
a. Communism which cannot tolerate multiplicity and freedom
b. Fascism which cannot allow dialogue
c. Democracy which requires centralistic political set up
d. Democratic socialism which advocates a genuine dialogue life
172. Which of the following statements about society is TRUE?
a. Revolution produce the opposite of their goals
b. Ethnic cleansing leads to social unanimity
c. “ Social education “ leads to a mature society
d. Cooperative education is a means to Social education?
173. Which of the following statements defines ethics?
a. The science of correct thinking
b. The study of being a general
c. The study of the nature of human knowledge
d. The study of rightness or wrongness of human knowledge
174. Which of the following dose not characterize fascism?
a. A government of laws and not of men
b. Rigid hierarchy and authority of single leader
c. Militarist, nationalist and dictatorial regime
d. It justifies any of its means by its end
175. Which of the following is not Anarchism?
a. Freedom and responsibility
b. Social organization without hierarchy
c. Extreme view of individual freedom
d. Destruction of the state and its supporting
institution
176. In which of the following acts would you attribute responsibility?
a. A barkeeper sells liquor to a minor not knowing he is an adult
b. A barkeeper sells liquor to a minor not knowing it is illegal to do so
c. A barkeeper sells liquor to a client not knowing it is contaminated with poison
d. A barkeeper sells liquor not knowing it would cause the death of a diabetic person
177. The values of the people are revealed in the following approaches, Except One
a. When they are asked to choose freely among alternative
b. When they act out what they choose but for a short while and not repeatedly
c. When they act out what they choose
d. When they are asked to choose among alternatives
178. To attain harmony in society people must rule their subjects with benevolence. whose teaching was
this?
a. Mohammad c. Buddha
b. Kung-fu-tzu d. Lao Tzu
179. The concept that school failed to encourage positive learning and were opposed to the production of
independently-minded adults was popularized by:
a. Paolo Freire c. Paul Goodman
b. John Dewey d. Ivan Illich
180. Which type of conscience is possessed by one who is extremely rigorous and is constantly afraid of
committing evil?
a. Scrupulous c. Certain conscience
b. Lax conscience d. Certain conscience
181. The following EXCEPT ONE are the strength of Filipino character
a. Pakikipagkapwa-tao c. Frankness
b. Family orientation d. Joy of humor
182. Which of the following is the Aristotlelian definition of happiness?
a. One’s ultimate achievements in life
b. A state made perfectly the aggregation of good things
c. The satisfaction of all desires and the absence of pain
d. The experience of the fulfillment of possibilities
183. One of the following statements about passion is NOT TRUE?
a. Passion does not affect voluntariness
b. Passion may completely destroy freedom
c. passion tends to blind the judgment of the intellect
d. passion cannot affect freedom
184. Which does not characterize a non-violent society?
a. Solves problems in non-adversarial way
b. Prefers indigenous ways of solving problems
c. A passive society
d. Solves problems in a non-confrontational way
185. Which statement holds true of values clarification?
a. Values are objective; no person has the right set of values to pass on to others
b. It is meant to help students get at their own feelings, ideas, and belief’s; no person has the right set of
values to pass on to others
c. No person has the right set of values to pass on to others; values are independent of time
d. Values are independent of time, place, and persons; meant to help students get at their own feelings and
ideas
186. Based on the Code of Ethics for teachers, which of the following may teachers Not do?
a. Welcomes the opportunity to lead in barangay
b. Studies and understands local customs and traditions
c. Deducts points from students scholastic ratings as a form of punishment
d. Attends church and worship of his choice joy and humor
187. Which type of justice implies the duty of one individual to give another what he is due?
a. International justice
b. Distributive justice
c. Social justice
d. Commutative justice
Direction: Choose the CORRECT answer with RATIONALE.
GOD BLESS Teachers!

1. Otherwise known as education for all season?


A. Formal Education
B. Non-formal Education
C. Informal Education
D. Distance Education
RATIONALE: Informal Education-can be acquired anytime and anywhere
Formal Education-hierarchy structured and chronologically graded; Elementary, Secondary, Tertiary
Non-formal Education- ALS
Distance Education-with the aid of computer technology, gadget, live session.

2. Career oriented education, is for?


A. Monasticist
B. Scholasticist
C. Chinese
D. Civalry
RATIONALE: Chinese- Civil Service Examination (Han Dynasty 206 BC)
Monasticist- religious discipline
Scholasticist-intellectual discipline
Chivalry-social discipline

3. They focus on practical and empirical education. Who are they?


A. Greeks
B. Chinese
C.Egyptian
D. Romans
RATIONALE: Egyptian-practical, empirical (Engineering,architecture, Mathematics, Geometry,
Astronomy, Medicine)
Greeks-liberal and democratic
Chinese-career oriented
Romans-pragmatic, progressive education

4. He established a school called Lyceum.


A. Socrates
B. Plato
C. Aristotle
D. Vespasian
RATIONALE: Aristotle-Lyceum
Socrates-Gnothi Seauton
Plato-Academy
Vespasian-founded Roman University and Roman Library

5. He legalized Christianity by virtue of Edict of Milan on AD 313.


A. Trajan
B. Pius
C. Constantine
D. Julian
RATIONALE: Constantine-legalized Christianity
Trajan-give scholars to poor but deserving students
Pius-exempted all Roman teacher from tax
Julian-elimated Christian teachers from Roman school

6. It is the school for new converts in early Christian Education.


A. Catechetical
B. Cathedral
C. Catechumenal
D. Episcopalian
RATIONALE: Catechumenal-school for new converts
Catechetical-school for leadership training
Cathedral or Episcopalian-shool for the clergy

7. It is education as a social discipline.


A. Guild
B. Scholasticism
C. Chivalry
D. Monasticism
RATIONALE:Chivalry-education for social discipline
Guild-education for middle class in Medieval Period
Monasticism-religious discipline

8. It stressed that individual freedom is prerequisite to the achievement of a rich and fulfilled life.
A. Reformation
B. Social Humanism
C. Italian Humanism
D. Northern Humanism
RATIONALE: Italian or Individual Humanism-freedom is prerequisite to be fulfilled in life
Reformation-protests of people in massive corruption in Roman Catholic Church
Social or Northern Humanism-education is a avenue for societal regeneration

9. He is the Father of Reformation.


A. Melanchton
B. Milton
C. Luther
D. Bacon
RATIONALE:Luther-Father of Reformation, wrote 95 theses denouncing Catholic Church
Melanchton-published the "Report or Book of Visitation, which is considered as the first educational
survey in history
Milton-boys should study formal grammar and formal education
Bacon-inductive method of teaching

10. He suggested that education should be made attractive rather than compulsive?
A. Comenius
B. Mulcaster
C. Rabelais
D. Montaigne
RATIONALE:Francois Rabelais-education should be attractive rather than compulsive
Comenius-pre-school system
Mulcaster-developed teacher training colleges (Normal schools)
Montaigne-field trip, education tours in the teaching
CURRICULUM DEVELOPMENT
Direction: Choose the CORRECT answer in a given questions.
NOTE: FOLLOW THE DIRECTION ☺😉😁 After an hour I will reveal the answer with
RATIONALE. 😋👍👌

1. Teacher Ybiel aims her students toward the holistic development. Teacher Ybiel is?
A. Progressivist
B. Essentialist
C. Idealist
D. Realist
RATIONALE: Progressivist-holistic development, consider ID, learner-centered, curriculum is flexible
Essentialist-value-centered
Idealist-preservation of one's freedom
Realist-actualities of life, face the real world, natural phenomena

2. The teachers have th right to teach students how to think but not what to think, this anchored in what
philosophy?
A. Idealism
B. Existentialism
C. Essentialism
D. Pragmatism
RATIONALE: Existentialism-enable man to make choices in life, teach students how to think but not to
what to think
Idealism-preservation of one's freedom
Essentialism- back to basics, value centered
Pragmatism-education must be useful to society, practical, beneficial

3. He introduced a curriculum for individual and societal development?


A. Erasmus
B. Da Feltre
C.Dewey
D. Luther
RATIONALE: Dewey-democratic education (curriculum for individual and social development)
Erasmus-liberal education, individualized instruction
Da Feltre-educate boys by used of games
Luther-reformation

4. It guided the child toward maturity within the context of social group.
A. Subject-Centered Curriculum
B. Child-Centered Curriculum
C. Experience Curriculum
D. Problem-Centered Curriculum
RATIONALE: Problem-centered-child maturity within the context of social group
Subject-centered-learner is expected to acquire
Child-centered-child is the center ot educative process
Experience-curriculum- immediate interest and needs of a child

5. Also called Area-of-Living Curriculum.


A. Core Curriculum
B. Learner-Centered Curriculum
C.Broad-fields Curriculum
D. Correlated Curriculum
RATIONALE: Core Curriculum-area of live curriculum
Learner-centered-experience and content around the life of child
Broad-fields curriculum-combined specific areas into larger fields
Correlated Curriculum-teaching similar topic into two or more subjects

6. It is a primary curricula renamed Social Science to Social Studies?


A. Primary Grade Curriculum of 1904
B. Primary Curriculum for SY 1936-1937
C. Primary Curriculum of 1913
D. Primary Curriculum of 1910
RATIONALE: Primary Grade Curriculum of 1904-3 grade for primary course; mental, physical, moral
Primary Curriculum 1913-GMRC
Primary Curriculum of 1910-industrial work for development of community
7. All of the following are the basic tool subjects, EXCEPT?
A. Filipino
B. Mathematics
C. MAPEH
D. English
RATIONALE: Basic tool subjects;English, Math, Filipino, Science

8. It is a learning area that serves as a "Laboratory of Life"?


A. TLE
B. Home Economics
C. Values Education
D. MAKABAYAN
RATIONALE: MAKABAYAN is general term, it includes; TLE, MAPEH, VALUES EDUCATION, SOCIAL
STUDIES
Home Economics is under TLE

9. It aims to prepare the child to face the real world?


A. Realism
B. Essentialism
C. Idealism
D. Pragmatism
RATIONALE: Realism-real world, actualities of life
Essentialism-basic
Idealism-anything in your mind (absolute true)
Pragmatism-pracical

10. It aims to develop the inherent power of the learner?


A. Realism
B. Reconstructionism
C. Pragmatism
D. Existentialism
RATIONALE: Reconstructionism-aims to develop inherent power of the learner, espoused a critical re-
examination, transform the society
Realism-actualities of life, real world
Pragmatism-practical
Existentialism-freedom of choice

Вам также может понравиться